Discuss how an automated answering system may be effectively utilized, and discuss the differences between this system and an answering service.  

ALL ANSWERS TO THESE QUESTIONS HAVE TO BE 200 WORDS IN LENGTH.

1.  Discuss how an automated answering system may be effectively utilized, and discuss the differences between this system and an answering service.

2.  Outline the major differences between new and established patients and how healthcare professionals should handle each patient type.
3.  Describe the pros and cons of using stream scheduling.

What five signs and symptoms are found in patients diagnosed with DKA that are not found in HHS?

2-3 SENTENCES EACH.

1) Answer the following questions related to abnormal assessment findings of the endocrine system.

a. Heat intolerance is associated with what secretion of the thyroid gland?

b. Striae are associated with excessive secretion of what gland?

c. Moon face is associated with the secretion of what of the adrenal gland?

d. Exophthalmos is associated with excessive secretion of what gland?

e. Hyperpigmentation is associated with hypofunction of what gland?

2) Determine whether each description is characteristic of type 1 or type 2 diabetes.

 

_Type 1 or Type 2

Description

Insulin resistance

 

Beta cell secretory exhaustion

 

Inherited defect in insulin receptors

 

Production of islet-cell antibodies

 

Genetic susceptibility for antibody production

3) What five signs and symptoms are found in patients diagnosed with DKA that are not found in HHS?

4) What is a thyroid storm and provide four clinical manifestations?

5) Name four effects aging have on the endocrine system.

a.

b.

c.

d.

6)  What are two primary laboratory findings used to confirm the diagnosis of hyperthyroidism and what test is used to differentiate Graves’ disease from other thyroid problems?

7)  Describe the difference between hyperparathyroidism and hypoparathyroidism. Provide a nursing intervention for each.

8) Describe the difference between hyperthyroidism and hypothyroidism. Provide a nursing intervention for each.

#2    ASSIGNMENT     Patient Profile: A 48-year-old female patient is scheduled for an incisional biopsy to evaluate a lump in her left breast. You are the nurse in the ambulatory care center where the procedure will take place.

Critical Thinking Questions: Answer each question completely and thoroughly.

1) How ill you describe the surgery to the patient when she arrives at the clinic?

2) If the biopsy is malignant, list three nursing diagnoses for this patient.

3) How could breast cancer treatment effect sex and intimacy for the patient?

4) What steps would you take when helping the patient decide what treatment to undergo for breast cancer?

5) How would you describe a modified radical mastectomy to the patient?

#3  ASSIGNMENT        Patient Profile: C. S., a 60-year-old truck driver, has been diabetic and has required insulin therapy for the past 12 years. Diabetic neuropathy has led to several complications, including end-stage renal disease, which he developed two years ago. He now receives hemodialysis three times a week and has an AV fistula in his left forearm. Yesterday, C. S. presented in the ED with a respiratory infection and was hospitalized with pneumonia, hypertension, and fluid overload. He will receive dialysis while on your nursing unit.

Critical Thinking Questions: Answer each question completely and thoroughly.

1) Is it appropriate to administer an aminoglycoside antibiotic for his respiratory infection? Why or why not?

2) What changes would you expect to see in this patient’s blood glucose levels during and after hemodialysis? What nursing care would you provide during and after hemodialysis?

3) C. S. missed his dialysis treatment on the day of admission. How might this be evidenced in his respiratory status? What care would you expect to provide next and why?

4) His wife comments on the fruity odor of his breath. What complication would cause this sign? What should you do next and why?

5) C.S. complains of dry, itching skin. How would you explain this symptom to him? What nursing measures for skin care would you provide and why?

#4   ASSIGNMENT   Oncology/Immunology CASE STUDY: 

Answer each question thoroughly and completely.

Save your name to this document

Patient Profile: C.W., a 35-year-old female patient, has just been diagnosed with systemic lupus erythematosus and is concerned about the impact of this multisystem disease process. You are the nurse that has been assigned to take care of this patient.

Critical Thinking Questions

1) What background information can you obtain from C.W. that would help you determine how the patient is reacting to the diagnosis?

2) When C.W. asks you what “multisystem disease process” means, how would you respond?

3) What diagnostic tests would serve to establish C.W.’s immune baseline?

4) What measures could you include with regard to discharge planning for this patient?

5) What are two nursing diagnosis for this patient and provide a rationale for each?

When you wake in the morning, you may reach for your cell phone to reply to a few text or email messages that you missed overnight. On your drive to work, you may stop to refuel your car. Upon your arrival, you might swipe a key card at the door to gain entrance to the facility. And before finally reaching your workstation, you may stop by the cafeteria to purchase a coffee.

From the moment you wake, you are in fact a data-generation machine. Each use of your phone, every transaction you make using a debit or credit card, even your entrance to your place of work, creates data. It begs the question: How much data do you generate each day? Many studies have been conducted on this, and the numbers are staggering: Estimates suggest that nearly 1 million bytes of data are generated every second for every person on earth.

As the volume of data increases, information professionals have looked for ways to use big data—large, complex sets of data that require specialized approaches to use effectively. Big data has the potential for significant rewards—and significant risks—to healthcare. In this Discussion, you will consider these risks and rewards.

To Prepare:

  • Review the Resources and reflect on the web article Big Data Means Big Potential, Challenges for Nurse Execs.
  • Reflect on your own experience with complex health information access and management and consider potential challenges and risks you may have experienced or observed.

Reflect on the role of a nurse leader as a knowledge worker.

Among the Resources in this module is the Rutherford (2008) article Standardized Nursing Language: What Does It Mean for Nursing Practice? In this article, the author recounts a visit to a local hospital to view the recent implementation of a new coding system.

During the visit, one of the nurses commented to her, “We document our care using standardized nursing languages but we don’t fully understand why we do” (Rutherford, 2008, para. 1).

How would you respond to a comment such as this one?

To Prepare:

  • Review the concepts of informatics as presented in the Resources, particularly Rutherford, M. (2008) Standardized Nursing Language: What Does It Mean for Nursing Practice?
  • Reflect on the role of a nurse leader as a knowledge worker.
  • Consider how knowledge may be informed by data that is collected/accessed.

The Assignment:

In a 2- to 3-page paper, address the following:

  • Explain how you would inform this nurse (and others) of the importance of standardized nursing terminologies.
  • Describe the benefits and challenges of implementing standardized nursing terminologies in nursing practice. Be specific and provide examples.
  • Be sure to support your paper with peer-reviewed research on standardized nursing terminologies that you consulted from the Walden Library.
  • 3 to 4 References  less than 5 years.

 How did the ideology of Manifest Destiny contribute to the Mexican War of 1846?

How did the ideology of Manifest Destiny contribute to the Mexican War of 1846? Why did the Mexican War of 1846 take place? Was the war necessary? Was it beneficial to the United States? What does this war tell us about this period of American history? Was it God’s plan that the United States extend from the Atlantic to the Pacific, or were imperialists looking for a way to defend what they planned to do anyway?

explain the Christian perspective of the nature of spirituality and ethics in contrast to the perspective of postmodern relativism within health care.

Based on the required topic study materials, write a reflection about worldview and respond to following:

  1. In 250-300 words, explain the Christian perspective of the nature of spirituality and ethics in contrast to the perspective of postmodern relativism within health care.
  2. In 250-300 words, explain what scientism is and describe two of the main arguments against it.
  3. In 750-1,000 words, answer each of the worldview questions according to your own personal perspective and worldview: (a) What is ultimate reality? (b) What is the nature of the universe? (c) What is a human being? (d) What is knowledge? (e) What is your basis of ethics? (f) What is the purpose of your existence?

Remember to support your reflection with the topic study materials.

While APA style is not required for the body of this assignment, solid academic writing is expected, and documentation of sources should be presented using APA formatting guidelines, which can be found in the APA Style Guide, located in the Student Success Center.

This assignment uses a rubric (LOCATED IN ATTACHMENTS). Please review the rubric prior to beginning the assignment to become familiar with the expectations for successful completion.

Based on the feedback offered by the provider, identify the best approach for teaching.

Based on the feedback offered by the provider, identify the best approach for teaching. Prepare a presentation based on the Teaching Work Plan and present the information to your community.

Options for Delivery

Select one of the following options for delivery and prepare the applicable presentation:

  1. PowerPoint presentation – no more than 30 minutes
  2. Pamphlet presentation – 1 to 2 pages
  3. Poster presentation

Selection of Community Setting

These are considered appropriate community settings. Choose one of the following:

  1. Public health clinic
  2. Community health center
  3. Long-term care facility
  4. Transitional care facility
  5. Home health center
  6. University/School health center
  7. Church community
  8. Adult/Child care center

Community Teaching Experience Approval Form

Before presenting information to the community, seek approval from an agency administrator or representative using the “Community Teaching Experience Approval Form.” Submit this form as directed in the Community Teaching Experience Approval assignment drop box.

General Requirements

While APA style is not required for the body of this assignment, solid academic writing is expected, and documentation of sources should be presented using APA formatting guidelines, which can be found in the APA Style Guide

Discussion: Organizational Policies and Practices to Support Healthcare Issues

Discussion: Organizational Policies and Practices to Support Healthcare Issues

Quite often, nurse leaders are faced with ethical dilemmas, such as those associated with choices between competing needs and limited resources. Resources are finite, and competition for those resources occurs daily in all organizations.

For example, the use of 12-hour shifts has been a strategy to retain nurses. However, evidence suggests that as nurses work more hours in a shift, they commit more errors. How do effective leaders find a balance between the needs of the organization and the needs of ensuring quality, effective, and safe patient care?

In this Discussion, you will reflect on a national healthcare issue and examine how competing needs may impact the development of polices to address that issue.

To Prepare:

· Review the Resources and think about the national healthcare issue/stressor you previously selected for study in Module 1.

· Reflect on the competing needs in healthcare delivery as they pertain to the national healthcare issue/stressor you previously examined.

Post an explanation of how competing needs, such as the needs of the workforce, resources, and patients, may impact the development of policy. Then, describe any specific competing needs that may impact the national healthcare issue/stressor you selected. What are the impacts, and how might policy address these competing needs? Be specific and provide examples.

Assignment: Developing Organizational Policies and Practices

Competing needs arise within any organization as employees seek to meet their targets and leaders seek to meet company goals. As a leader, successful management of these goals requires establishing priorities and allocating resources accordingly.

Within a healthcare setting, the needs of the workforce, resources, and patients are often in conflict. Mandatory overtime, implementation of staffing ratios, use of unlicensed assisting personnel, and employer reductions of education benefits are examples of practices that might lead to conflicting needs in practice.

Leaders can contribute to both the problem and the solution through policies, action, and inaction. In this Assignment, you will further develop the white paper you began work on in Module 1 by addressing competing needs within your organization.

To Prepare:

· Review the national healthcare issue/stressor you examined in your Assignment for Module 1, and review the analysis of the healthcare issue/stressor you selected.

· Identify and review two evidence-based scholarly resources that focus on proposed policies/practices to apply to your selected healthcare issue/stressor.

· Reflect on the feedback you received from your colleagues on your Discussion post regarding competing needs.

The Assignment (4-5 pages):

Developing Organizational Policies and Practices

Add a section to the paper you submitted in Module 1. The new section should address the following:

· Identify and describe at least two competing needs impacting your selected healthcare issue/stressor.

· Describe a relevant policy or practice in your organization that may influence your selected healthcare issue/stressor.

· Critique the policy for ethical considerations, and explain the policy’s strengths and challenges in promoting ethics.

· Recommend one or more policy or practice changes designed to balance the competing needs of resources, workers, and patients, while addressing any ethical shortcomings of the existing policies. Be specific and provide examples.

· Cite evidence that informs the healthcare issue/stressor and/or the policies, and provide two scholarly resources in support of your policy or practice recommendations.

Please Use the resources below for reference. 

American Nurses Association. (2015). Code of ethics for nurses with interpretive statements. Silver Spring, MD: Author. Retrieved from https://www.nursingworld.org/coe-view-only

Note: Review all, with special attention to “Provision 6” (pp. 23–26).

Kelly, P., & Porr, C. (2018). Ethical nursing care versus cost containment: Considerations to enhance RN practice. OJIN: Online Journal of Issues in Nursing, 23(1), Manuscript 6. doi:10.3912/OJIN.Vol23No01Man06. Retrieved from http://ojin.nursingworld.org/MainMenuCategories/ANAMarketplace/ANAPeriodicals/OJIN/TableofContents/Vol-23-2018/No1-Jan-2018/Ethical-Nursing-Cost-Containment.html

Milliken, A. (2018). Ethical awareness: What it is and why it matters. OJIN: Online Journal of Issues in Nursing, 23(1), Manuscript 1. doi:10.3912/OJIN.Vol23No01Man01. Retrieved from http://ojin.nursingworld.org/MainMenuCategories/ANAMarketplace/ANAPeriodicals/OJIN/TableofContents/Vol-23-2018/No1-Jan-2018/Ethical-Awareness.html

Students will use the information from the earlier PICOT Statement Paper and Literature Evaluation Table assignments to develop a 750-1,000 word review that includes the following sections

While the implementation plan prepares students to apply their research to the problem or issue they have identified for their capstone change proposal project, the literature review enables students to map out and move into the active planning and development stages of the project.

A literature review analyzes how current research supports the PICOT, as well as identifies what is known and what is not known in the evidence. Students will use the information from the earlier PICOT Statement Paper and Literature Evaluation Table assignments to develop a 750-1,000 word review that includes the following sections:

  1. Title page
  2. Introduction section
  3. A comparison of research questions
  4. A comparison of sample populations
  5. A comparison of the limitations of the study
  6. A conclusion section, incorporating recommendations for further research

Prepare this assignment according to the guidelines found in the APA Style Guide, located in the Student Success Center. An abstract is not required.

This assignment uses a rubric. Please review the rubric prior to beginning the assignment to become familiar with the expectations for successful completion.

You are required to submit this assignment to LopesWrite

Doctoral Project Proposal Checklist School Level Review

Doctoral Project Proposal Checklist School Level Review

School of Nursing and Health Sciences

Instructions

Please use the DPP School Level Review Checklist to ensure mentees have met the requirements within each section of the DPP form. This checklist is a guide to help the mentor and the mentee.

1. It is an expectation that the mentee will review the checklist and initial all items completed prior to sending to mentor, found at: https://campus.capella.edu/web/doctoral-programs/professional//capstone or

https://campus.capella.edu/web/school-of-nursing-and-health-sciences/doctoral-capstone   

2. Mentor review of the DPP checklist:

a. The mentor will review each section of the DPP checklist confirming the outlined requirements have been met.

b. If met, the mentor will place their initials in the met box for that section.

c. If not met, the mentor will place their initials in the not met box and provide feedback under “Reviewer’s Comments” in the sections not met.

3. Committee review of the DPP checklist:

a. Once the mentor has approved the DPP form and completed a review of the checklist, the mentor will send the initialed DPP checklist and the DPP form to the mentee’s committee for review.

Each committee member will

a. If met, place their initials in the met box for that section.

b. If not met, place their initials in the not met box and provide feedback under “Reviewer’s Comments” in the sections not met.

c. The committee member will send the checklist back to the mentor to provide to the mentee.

4. School level review- Once ready for school level review, the mentor will:

a. Remove all comments on the DPP checklist that have been corrected

b. Leave the reviewer’s initials approving the DPP form as met.

c. Send the DPP checklist with the approved DPP form to school level review.

5. School level review will use the DPP checklist to confirm that each section has been met. If not approved the reviewers will provide feedback on how to meet the requirements on the DPP checklist under “Reviewer comments” within each section.

Learner__________________________________ Mentor ___________________________________________

Committee Members _______________________________________________________________________________

Section Requirements : DPP Part 1 Met Not Met / Comments
PICOT Question · Appropriate PICOT question that meets all components to include population, intervention, comparison (If applicable), outcome, and time.

Reviewer’s comments to learner:

1.

Demining a Need and Gap in Practice · Explains the background and significance of the proposed practice problem with the organization.

· Provides internal data to support that this problem is a priority through a needs assessment within the practicum site.

· Discusses the current practice for the practice problem, such as the current the policy/procedure or what is currently done.

· Include description of conversations with leadership and/or key decision makers, as well as stakeholders who will be impacted by the project.

· Provide 2-4 studies or EBP guidelines that support this needed change

Reviewer’s comments to learner:

·

Population/Problem · Describes the population and problem in terms of role, age range, diagnosis, procedure, etc.

Reviewer’s comments to learner:

1.

Intervention · List 2-3 potential interventions that can be applied in this practice change?

· Practice recommendations are identified and consistent with the synthesis of the literature.

· There is logical progression from the synthesis of the literature to the practice recommendation to the proposed intervention of the project.

· Defines the intervention very specifically and operationally so that anyone replicating this project could apply the same intervention.

· Describes the learner role in the practice change

· Appropriate appendices are included

Reviewer’s comments to learner:

1.

Comparison · Discusses the comparison for the intervention which is likely the standard treatment or usual care.

· If there is no comparison, the learner has discussed the current state versus the desired state of care.

· Discusses the potential impact of the practice change to which that the learner will be comparing.

Reviewer’s comments to learner:

1.

Outcomes · Discusses the desirable outcome(s).

· The outcomes are measurable. Describes how the outcome(s) will be measured.

· Describes the validity and reliability of any existing tool that they will use.

· Describes at what point(s) in time the data will be collected and how it will be analyzed.

· Measures compliance by discussing what evaluation criteria will use to ensure the planned change occurred as planned.

· Discusses the type of data produced by the evaluation tools (e.g., nominal, ordinal or interval). Discusses what statistical tests will be used to determine if the selected intervention produced the outcome desired.

Reviewer’s comments to learner:

1.

Timeframe · Provides a detailed and feasible work plan and timeline for completing the capstone. The timeframe is broken down by milestones within each quarter.

Reviewer’s comments to learner:

1.

School level review performed by and date:

Version 1 __________________________________________________________

Version 2 __________________________________________________________

Version 3 __________________________________________________________

Version 3 co-reviewer ________________________________________________

Version 4 __________________________________________________________

Learner__________________________________ Mentor ___________________________________________

Committee Members _______________________________________________________________________________

Section Requirements: DPP Part 2 Met Not Met/ Comments
Introduction · Identify the focus of the project and the significance (includes at least 1-2 background studies).

· Provides the project goals and the relationship to the PICOT question.

Reviewer’s comments to learner:

1.

Review of Literature · Articles reviewed with a minimum of 20 required, all published within the last five years, unless a seminal article.

· Describes extensive review of the literature (including keywords, search phrases, databases used, number of articles found, inclusion/exclusion criteria, number of articles retained and why).

· Provides information on the significance of the problem, such as the following: incidence and/or prevalence, national data and quality/regulatory expectations, impact on patient, family, and healthcare system, financial implications, cultural, healthcare policy, quality, safety, legal, ethical implications if pertinent.

· Thorough review and critical analysis of literature.

· Synthesis of evidence supporting the question and project. This is not an annotated bibliography.

· Provides completed evidence table.

Reviewer’s comments to learner:

1.

Framework · Describes theoretical framework or conceptual model.

· Connects study goals and PICOT to framework or model.

· Defines study variables.

· Describes any study assumptions.

Reviewer’s comments to learner:

1.

Method · Describes project design, setting, population and sampling.

· Describes human subject’s protection methods (as applicable).

· Identifies outcome measures.

· Describes implementation plans including thorough description of implementation process; psychometric properties (reliability and validity) of any instruments used; status of permission to use instruments; procedures for data collection.

· If the instrument is not public, permission to use the instrument is attached as an appendix.

· Describes project evaluation including type of data analysis.

· Discusses the anticipated findings.

· Discusses the study limitations:

· Factors that might limit internal validity.

What efforts will be taken to minimize and or adjust for the limitations?

Reviewer’s comments to learner:

1.

Sampling · Discusses the inclusion and exclusion criteria.

Reviewer’s comments to learner:

1.

Meet program-specific requirements for proposal/research plan approval. · Completes evidence table, includes all references in DPP.

· Complete reference list for DPP.

Reviewer’s comments to learner:

1.

Write with logic, flow, etc.
Write with proper grammar, usage, APA etc.
Date of learner revision(s)
Date of preceptor revision(s) request/approval
Date of committee revision(s) request/approval

School level review performed by and date:

Version 1 __________________________________________________________

Version 2 __________________________________________________________

Version 3 __________________________________________________________

Version 3 co-reviewer ________________________________________________

Version 4 __________________________________________________________

December 7, 2017

Explain how social determinants of health contribute to the development of disease.  

What are social determinants of health?  Explain how social determinants of health contribute to the development of disease.  Describe the fundamental idea that the communicable disease chain model is designed to represent. Give an example of the steps a nurse can take to break the link within the communicable disease chain.

Discuss how health care delivery systems work collaboratively to address global health concerns and some of the stakeholders that work on these issues.

Select a global health issue affecting the international health community. Briefly describe the global health issue and its impact on the larger public health care systems (i.e., continents, regions, countries, states, and health departments). Discuss how health care delivery systems work collaboratively to address global health concerns and some of the stakeholders that work on these issues.

Consider the impact that EBP may have on factors impacting  these quadruple aim elements, such as preventable medical errors or  healthcare delivery.

To Prepare:

  • Read the articles by Sikka, Morath, & Leape (2015);  Crabtree, Brennan, Davis, & Coyle (2016); and Kim et al. (2016)  provided in the Resources.
  • Reflect on how EBP might impact (or not impact) the Quadruple Aim in healthcare.
  • Consider the impact that EBP may have on factors impacting  these quadruple aim elements, such as preventable medical errors or  healthcare delivery.

To Complete:

Write a brief analysis (no longer than 2 pages) of the connection between EBP and the Quadruple Aim.

Your analysis should address how EBP might (or might not) help reach the Quadruple Aim, including each of the four measures of:

  • Patient experience
  • Population health
  • Costs
  • Work life of healthcare providers

    The Quadruple Aim: care, health, cost and meaning in work

    Rishi Sikka,1 Julianne M Morath,2 Lucian Leape3

    1Advocate Health Care, Downers Grove, Illinois, USA 2Hospital Quality Institute, Sacramento, California, USA 3Harvard School of Public Health, Boston, Massachusetts, USA

    Correspondence to Dr Rishi Sikka, Advocate Health Care, 3075 Highland Avenue, Suite 600, Downers Grove, Il 60515, USA; rishi.sikka@advocatehealth.com

    Received 5 March 2015 Revised 6 May 2015 Accepted 16 May 2015

    To cite: Sikka R, Morath JM, Leape L. BMJ Qual Saf 2015;24:608–610.

    In 2008, Donald Berwick and colleagues provided a framework for the delivery of high value care in the USA, the Triple Aim, that is centred around three over- arching goals: improving the individual experience of care; improving the health of populations; and reducing the per capita cost of healthcare.1 The intent is that the Triple Aim will guide the redesign of healthcare systems and the transition to population health. Health systems glo- bally grapple with these challenges of improving the health of populations while simultaneously lowering healthcare costs. As a result, the Triple Aim, although ori- ginally conceived within the USA, has been adopted as a set of principles for health system reform within many organi- sations around the world. The successful achievement of the

    Triple Aim requires highly effective healthcare organisations. The backbone of any effective healthcare system is an engaged and productive workforce.2 But the Triple Aim does not explicitly acknow- ledge the critical role of the workforce in healthcare transformation. We propose a modification of the Triple Aim to acknow- ledge the importance of physicians, nurses and all employees finding joy and meaning in their work. This ‘Quadruple Aim’ would add a fourth aim: improving the experience of providing care. The core of workforce engagement is

    the experience of joy and meaning in the work of healthcare. This is not synonym- ous with happiness, rather that all members of the workforce have a sense of accomplishment and meaning in their contributions. By meaning, we refer to the sense of importance of daily work. By joy, we refer to the feeling of success and fulfilment that results from meaning- ful work. In the UK, the National Health Service has captured this with the notion of an engaged staff that ‘think and act in a positive way about the work they do, the people they work with and the organ- isation that they work in’.3

    The evidence that the healthcare work- force finds joy and meaning in work is not encouraging. In a recent physician survey in the USA, 60% of respondents indicated they were considering leaving practice; 70% of surveyed physicians knew at least one colleague who left their practice due to poor morale.2 A 2015 survey of British physicians reported similar findings with approximately 44% of respondents reporting very low or low morale.4 These findings also extend to the nursing profession. In a 2013 US survey of registered nurses, 51% of nurses worried that their job was affect- ing their health; 35% felt like resigning from their current job.5 Similar findings have been reported across Europe, with rates of nursing job dissatisfaction ranging from 11% to 56%.6

    This absence of joy and meaning experi- enced by a majority of the healthcare workforce is in part due to the threats of psychological and physical harm that are common in the work environment. Workforce injuries are much more frequent in healthcare than in other industries. For some, such as nurses’ aides, orderlies and attendants, the rate is four times the indus- trial average.7 More days are lost due to occupational illness and injury in health- care than in mining, machinery manufac- turing or construction.7

    The risk of physical harm is dwarfed by the extent of psychological harm in the complex environment of the health- care workplace. Egregious examples include bullying, intimidation and phys- ical assault. Far more prevalent is the psy- chological harm due to lack of respect. This dysfunction is compounded by pro- duction pressure, poor design of work flow and the proportion of non-value added work. The current dysfunctional healthcare

    work environment is in part a by-product of the gradual shift in healthcare from a public service to a business model that occurred in the latter half of the 20th

    EDITORIAL

    608 Sikka R, et al. BMJ Qual Saf 2015;24:608–610. doi:10.1136/bmjqs-2015-004160

    century.8 Complex, intimate caregiving relationships have been reduced to a series of transactional demand- ing tasks, with a focus on productivity and efficiency, fuelled by the pressures of decreasing reimbursement. These forces have led to an environment with lack

    of teamwork, disrespect between colleagues and lack of workforce engagement. The problems exist from the level of the front-line caregivers, doctors and nurses, who are burdened with non-caregiving work, to the healthcare leader with bottom-line worries and disproportionate reporting requirements. Without joy and meaning in work, the workforce cannot perform at its potential. Joy and meaning are generative and allow the best to be contributed by each individual, and the teams they comprise, towards the work of the Triple Aim every day. The precondition for restoring joy and meaning is

    to ensure that the workforce has physical and psycho- logical freedom from harm, neglect and disrespect. For a health system aspiring to the Triple Aim, fulfill- ing this precondition must be a non-negotiable, endur- ing property of the system. It alone does not guarantee the achievement of joy and meaning, however the absence of a safe environment guarantees robbing people of joy and meaning in their work. Cultural freedom from physical and psychological harm is the right thing to do and it is smart economics because toxic environments impose real costs on the organisation, its employees, physicians, patients and ultimately the entire population. An organisation focused on enabling joy and

    meaning in work and pursuit of the Triple Aim needs to embody shared core values of mutual respect and civility, transparency and truth telling and the safety of the workforce. It recognises the work and accom- plishments of the workforce regularly and with high visibility. For the individual, these notions of joy and meaning in healthcare work are recognised in three critical questions posed by Paul O’Neill, former chair- man and chief executive officer of Alcoa. This is an internal gut-check, that needs to be answered affirma- tively by each worker each day:2

    1. Am I treated with dignity and respect by everyone, everyday, by everyone I encounter, without regard to race, ethnicity, nationality, gender, religious belief, sexual orientation, title, pay grade or number of degrees?

    2. Do I have the things I need: education, training, tools, financial support, encouragement, so I can make a con- tribution this organisation that gives meaning to my life?

    3. Am I recognised and thanked for what I do? If each individual in the workforce cannot answer

    affirmatively to these questions, the full potential to achieve patient safety, effective outcomes and lower costs is compromised. The leadership and governance of our healthcare

    systems currently have strong economic and outcome motivations to focus on the Triple Aim. They also need to feel a parallel moral obligation to the

    workforce to create an environment that ensures joy and meaning in work. For this reason, we recommend adding a fourth essential aim: improving the experi- ence of providing care. The notion of changing the objective to the Quadruple Aim recognises this focus within the context of the broader transformation required in our healthcare system towards high value care. While the first three aims provide a rationale for the existence of a health system, the fourth aim becomes a foundational element for the other goals to be realised. Progress on this fourth goal in the Quadruple Aim

    can be measured through metrics focusing on two broad areas: workforce engagement and workforce safety. Workforce engagement can be assessed through annual surveys using established frameworks that allow for benchmarking within industry and with non-healthcare industries.9 Measures should also be extended to quantify the opposite of engagement, workforce burn-out. This could include select ques- tions from the Maslach Burnout Inventory, the gold standard for measuring employee burn-out.10 In the realm of workforce safety, metrics should include quantifying work-related deaths or disability, lost time injuries, government mandated reported injuries and all injuries. Although these measures do not com- pletely quantify the experience of providing care, they provide a practical start that is familiar and allow for an initial baseline assessment and monitoring for improvement. The rewards of the Quadruple Aim, achieved within

    an inspirational workplace could be immense. No other industry has more potential to free up resources from non-value added and inefficient production practices than healthcare; no other industry has more potential to use its resources to save lives and reduce human suffering; no other industry has the potential to deliver the value envisioned by The Triple Aim on such an audacious scale. The key is the fourth aim: creating the conditions for the healthcare workforce to find joy and meaning in their work and in doing so, improving the experience of providing care.

    Contributors All authors assisted in the drafting of this manuscript.

    Competing interests None declared.

    Provenance and peer review Not commissioned; externally peer reviewed.

    REFERENCES 1 Berwick DM, Nolan TW, Whittington J. The triple aim: care,

    health and cost. Health Aff 2008;27:759–69. 2 Lucian Leape Institute. 2013. Through the eyes of the

    workforce: creating joy, meaning and safer health care. Boston, MA: National Patient Safety Foundation.

    3 NHS employers staff engagement. http://www.nhsemployers. org/staffengagement (accessed 4 May 2015).

    4 BMA Quarterly Tracker Survey. http://bma.org.uk/working- for-change/policy-and-lobbying/training-and-workforce/

    Editorial

    Sikka R, et al. BMJ Qual Saf 2015;24:608–610. doi:10.1136/bmjqs-2015-004160 609

    tracker-survey/omnibus-survey-january-2015 (accessed 4 May 2015).

    5 AMN Healthcare 2013 survey of registered nurses. http://www. amnhealthcare.com/uploadedFiles/MainSite/Content/ Healthcare_Industry_Insights/Industry_Research/2013_ RNSurvey.pdf (accessed 4 May 2015).

    6 Aiken LH, Sermeus W, Van Den HeedeKoen, et al. Patient safety, satisfaction and quality of hospital care: cross sectional surveys of nurses and patients in 12 countries in Europe and the United States. BMJ 2012;344:e1717.

    7 US Department of Labor Bureau of Labor Statistics. Occupational injuries and illnesses (annual) news release.

    Workplace injuries and illnesses 2009. 21 October 2010. http://www.bls.gov/news.release/archives/osh_10212010.htm (accessed 4 May 2015).

    8 Morath J. The quality advantage, a strategic guide for health care leaders. AHA Press, 1999:225.

    9 Surveys on Patient Safety Culture. Agency for Healthcare Research and Quality. http://www.ahrq.gov/professionals/quality- patient-safety/patientsafetyculture/index.html (accessed 4 May 2015).

    10 Maslach C, Jackson S, Leiter M. Maslach burnout inventory manual. 3rd edn. Palo Alto, CA: Consulting Psychologists Press, 1996.

    Editorial

    610 Sikka R, et al. BMJ Qual Saf 2015;24:608–610. doi:10.1136/bmjqs-2015-004160

    • The Quadruple Aim: care, health, cost and meaning in work
      • References

Describe the organization’s overall readiness based on your findings.

Research a health care organization or network that spans several states with in the United States (United Healthcare, Vanguard, Banner Health, etc.). Assess the readiness of the health care organization or network you chose in regard to meeting the health care needs of citizens in the next decade.

Prepare a 1,000-1,250 word paper that presents your assessment and proposes a strategic plan to ensure readiness. Include the following:

  1. Describe the health care organization or network.
  2. Describe the organization’s overall readiness based on your findings.
  3. Prepare a strategic plan to address issues pertaining to network growth, nurse staffing, resource management, and patient satisfaction.
  4. Identify any current or potential issues within the organizational culture and discuss how these issues may affect aspects of the strategic plan.
  5. Propose a theory or model that could be used to support implementation of the strategic plan for this organization. Explain why this theory or model is best.

Prepare this assignment according to the guidelines found in the APA Style Guide, located in the Student Success Center. An abstract is not required.

This assignment uses a rubric. Please review the rubric prior to beginning the assignment to become familiar with the expectations for successful completion.

The wife and daughter need teaching about his antiepileptic medication. What teaching should be included?

An older client was recently discharged from the hospital for evaluation of seizure activity. His history reveals that he has late-stage Alzheimer’s disease, Parkinson’s disease, hypertension, and type II diabetes mellitus, which is controlled by diet. He lives at home, where his wife and daughter take care of him. His discharge medications include phenytoin (Dilantin), 100 mg BID; hydrochlorothiazide (HydroDIURIL), 50 mg QD; levodopa (Sinemet), 25/100 TID; and haloperidol (Haldol), 1 mg before bed. The client has been referred for home care nursing follow-up.

Questions:

  1. On the initial home visit by the nurse, what assessments should be made?
  2. The wife and daughter need teaching about his antiepileptic medication. What teaching should be included?
  3. During the initial home visit, the client experiences a generalized seizure. What action should the nurse take?

In this Assignment, you will analyze recent presidential healthcare agendas.

It may seem to you that healthcare has been a national topic of debate among political leaders for as long as you can remember.

Healthcare has been a policy item and a topic of debate not only in recent times but as far back as the administration of the second U.S. president, John Adams. In 1798, Adams signed legislation requiring that 20 cents per month of a sailor’s paycheck be set aside for covering their medical bills. This represented the first major piece of U.S. healthcare legislation, and the topic of healthcare has been woven into presidential agendas and political debate ever since.

As a healthcare professional, you may be called upon to provide expertise, guidance and/or opinions on healthcare matters as they are debated for inclusion into new policy. You may also be involved in planning new organizational policy and responses to changes in legislation. For all of these reasons you should be prepared to speak to national healthcare issues making the news.

In this Assignment, you will analyze recent presidential healthcare agendas. You also will prepare a fact sheet to communicate the importance of a healthcare issue and the impact on this issue of recent or proposed policy.

To Prepare:

  • Review the agenda priorities of the current/sitting U.S. president and the two previous presidential administrations.
  • Select an issue related to healthcare that was addressed by each of the last three U.S. presidential administrations.
  • Reflect on the focus of their respective agendas, including the allocation of financial resources for addressing the healthcare issue you selected.
  • Consider how you would communicate the importance of a healthcare issue to a legislator/policymaker or a member of their staff for inclusion on an agenda.

The Assignment: (1- to 2-page Comparison Grid, 1-Page Analysis, and 1-page Fact Sheet)

Part 1: Agenda Comparison Grid

Use the Agenda Comparison Grid Template found in the Learning Resources and complete the Part 1: Agenda Comparison Grid based on the current/sitting U.S. president and the two previous presidential administrations and their agendas related to the public health concern you selected. Be sure to address the following:

  • Identify and provide a brief description of the population health concern you selected and the factors that contribute to it.
  • Describe the administrative agenda focus related to the issue you selected.
  • Identify the allocations of financial and other resources that the current and two previous presidents dedicated to this issue.
  • Explain how each of the presidential administrations approached the issue.

(A draft of Part 1: Agenda Comparison Grid should be posted to the Module 1 Discussion Board by Day 3 of Week 1.)

Part 2: Agenda Comparison Grid Analysis

Using the information you recorded in Part 1: Agenda Comparison Grid on the template, complete the Part 2: Agenda Comparison Grid Analysis portion of the template, by addressing the following:

  • Which administrative agency would most likely be responsible for helping you address the healthcare issue you selected?
  • How do you think your selected healthcare issue might get on the agenda for the current and two previous presidents? How does it stay there?
  • Who would you choose to be the entrepreneur/ champion/sponsor of the healthcare issue you selected for the current and two previous presidents?

Part 3: Fact Sheet or Talking Points Brief

Based on the feedback that you received from your colleagues in the Discussion, revise Part 1: Agenda Comparison Grid and Part 2: Agenda Comparison Grid Analysis.

Then, using the information recorded on the template in Parts 1 and 2, develop a 1-page Fact Sheet or Talking Points Brief that you could use to communicate with a policymaker/legislator or a member of their staff for this healthcare issue. You can use Microsoft Word or PowerPoint to create your Fact Sheet or Talking Point Brief. Be sure to address the following:

  • Summarize why this healthcare issue is important and should be included in the agenda for legislation.
  • Justify the role of the nurse in agenda setting for healthcare issues.

By Day 3 of Week 1

Submit a draft of Part 1: Agenda Comparison Grid to the Module 1 Discussion Board

Considering the feedback you received in the Module 1 Discussion on Part 1: Agenda Comparison Grid, revise and further develop Part 1: Agenda Comparison Grid and Part 2: Agenda Comparison Grid Analysis. Consider how the feedback you received might influence your Fact Sheet or Talking Points and revise as necessary.

By Day 7 of Week 2

Submit your final version of Part 1: Agenda Comparison Grid, Part 2: Agenda Comparison Grid Analysis, and Part 3: Fact Sheet or Talking Points Brief.

What are the major ethical, societal, professional, and legal system-level issues confronting providers, insurers, public policymakers, and organizations regarding the care for this population

Among various stakeholders, the registered nurse is an advocate for patients, their health and the care delivered to them. There is a need to evaluate the health status of vulnerable populations, to assess nursing’s role in health initiatives, and to find ways that we can help improve health. This presentation enables students to assess the role of organized efforts to influence health care delivery, and the contributions of medical technology, research findings, and societal values on our evolving health care delivery system.

This presentation will examine the role of the nurse as a health professional and leader.

1.      Choose a high-risk risk population

2.      Address a health care need for this population

3.      What resources are provided for this population

4.      What are the economic priorities, challenges, and issues for this population

5.      What are the major ethical, societal, professional, and legal system-level issues confronting providers, insurers, public policymakers, and organizations regarding the care for this population

6.      How could nursing intervene or advocate to improve the health care delivery for this population

This PowerPoint® (Microsoft Office) or Impress® (Open Office) presentation should be a minimum of 20 slides, including a title and reference slide, with detailed speaker notes on content slides and recorded audio comments for Introduction, Recommendations, and Conclusion slides.

Here is another resource to help you to add audio to the introduction, recommendation and conclusion slides: https://support.office.com/en-us/article/record-a-slide-show-with-narration-and-slide-timings-0b9502c6-5f6c-40ae-b1e7-e47d8741161c

Please practice doing this before the due date.

Use at least four scholarly sources and make certain to review the module’s Signature Assignment Rubric before starting your presentation. This presentation is worth 400 points for quality content and presentation.

Sultz, H. A. & Young, K. M. (2011). Health Care USA: Understanding its organization and delivery.  In T. Reilly (Eds.). Sudbury, MA: Jones & Bartlett Learning.

Compare and contrast the quantitative and qualitative research article that you choose for the EBP topic

Signature Assignment Title: Theory, Research, and Practice in Nursing

Signature Assignment Description/Directions: Presentation

A nurse on the research and practice committee is preparing for an update on evidence-based practice (EBP). There is a need to address the potential changes with several EBP policies. This presentation will examine the EBP policies as well as reasoning and rationale as to why changes may be needed.

  • Choose an EBP topic and two scholarly peer-reviewed journal articles
  • Compare the difference between theory, research, and practice in nursing
  • Choose a theory that best correlates with the EBP practice change that you would like to make
  • Compare and contrast the quantitative and qualitative research article that you choose for the EBP topic
  • What technology did you use to locate the articles? Databases? Search terms?
  • What are the philosophical, theoretical, and methodological perspectives in the research articles that were chosen?
  • What are the ethical and scientific integrity issues related to the research?
  • How did your nursing knowledge advance through the utilization of research?

This PowerPoint® (Microsoft Office) or Impress® (Open Office) presentation should be a minimum of 20 slides, including a title, introduction, conclusion and reference slide, with detailed speaker notes and recorded audio comments for all content slides. Use at least four scholarly sources and make certain to review the module’s Signature Assignment Rubric before starting your presentation. This presentation is worth 400 points for quality content and presentation.

Total Point Value of Signature Assignment: 400 points

Tappen, R. M. (2016). Advanced nursing research: From theory to practice (2nd ed.). Burlington, MA: Jones & Bartlett Learnin

Describe the difference between a nursing practice problem and a medical practice problem

PICOT is utilized by the health care community to identify and study a nursing or medical practice problem. Consequently, PICOT examples that may provide insight into the use of the PICOT process, may not be relevant to nursing practice as they are based on a medical practice problem.
Describe the difference between a nursing practice problem and a medical practice problem. Provide one example of each. Discuss why is it important to ensure your PICOT is based on a nursing practice problem.

https://journals.lww.com/nursing/fulltext/2014/02000/To_make_your_case,_start_with_a_PICOT_question.7.aspx

Successful completion of a doctoral dissertation requires significant amounts of independent reading on the research topic.

Successful completion of a doctoral dissertation requires significant amounts of independent reading on the research topic. This allows the doctoral learner/researcher to become familiar with the scope of the topic and to identify gaps or tensions within the existing literature on the topic. These gaps and tensions become the source of the dissertation research. In this assignment, you will read and annotate potential sources in your dissertation field of interest. Those demonstrating the most merit to the best of your understanding of the topic at this time should be added to your RefWorks list for potential inclusion in the literature review section of your dissertation.

General Requirements:

Use the following information to ensure successful completion of the assignment:

  • It is strongly recommended that you engage in this activity throughout the duration of your program. You will be adding to this document to begin building a runnnig literature review. You will have five sources in your list by the end of this assignment.
  • The College of Doctoral Studies recognizes the diversity of learners in our programs and the varied interests in research topics for their dissertations in the Social Sciences.

    Dissertation topics must, at a minimum, be aligned to General Psychology in the Ph.D. program, Leadership in the Ed.D. Organizational Leadership program, Adult Instruction in the Ed.D. Teaching and Learning program, Management in the DBA program, and Counseling Practice, Counselor Education, Clinical Supervision or Advocacy/Leadership within the Counseling field in the Counselor Education Ph.D. program.

    If there are questions regarding appropriate alignment of a dissertation topic to the program, the respective program chair will be the final authority for approval decisions.

    Specifically, although the College prefers a learner’s topic align with the program emphasis, this alignment is not “required.” The College will remain flexible on the learner’s dissertation topic if it aligns with the degree program in which the learner is enrolled. The Ph.D. program in General Psychology does not support clinically based research.

  • Instructors will be scoring your submission based on the number of unique sources identified in the list submitted.
  • Download the resource Literature Review Resources Tool and use it to complete the assignment.
  • Doctoral learners are required to use APA style for their writing assignments. The APA Style Guide is located in the Student Success Center.
  • Refer to the resource, “Preparing Annotated Bibliographies,” located in the Student Success Center, for additional guidance on completing this assignment in the appropriate style.
  • You are required to submit this assignment to LopesWrite. Refer to the directions in the Student Success Center.

Directions:

Read at least five empirical articles in your general dissertation field.

In the “Literature Review Resources” document, provide the following for each source:

  1. The APA formatted citation.
  2. A brief annotation of the key points of the source.
  3. An indication of whether the source has been added to (Y) or excluded from (N) your RefWorks list.Running head: SHORTENED TITLE 1

    TITLE GOES HERE 2

    Title Should Suggest the Topic

    Your Name

    Grand Canyon University

    <Date>

    Literature Review Resources

    One of the most important aspects of your research will be how you organize your resources. RefWorks is an optional citation management software tool you can sign up to use that helps organize your citations. To find out more about how to sign up for your free, optional RefWorks account, review the information here:

    http://libguides.gcu.edu/refworksandendnote.

    Number Article Information: Be sure entries are presented in alphabetical order. Added to RefWorks? Y or N (optional)

    1. Reference
    Permalink
    Annotation
    2. Reference
    Permalink
    Annotation
    3. Reference
    Permalink
    Annotation
    4. Reference
    Permalink
    Annotation
    5. Reference
    Permalink
    Annotation

Please, respond to the following discussion by using one reference from peer-reviewed Nursing Journal not older than 5 years.

Please, respond to the following discussion by using one reference from peer-reviewed Nursing Journal not older than 5 years. APA required without cover page. (one paragraph)

 

The relationship between national and global health and nursing leadership is extremely important. Health equity is essential for people to have the opportunity to live productive and healthy lives and achieve their full potential (Huber, 2018). Sadly, in the global and national health arenas, this is not so, and there are grave disparities in access to healthcare. The nurse leader has a duty and responsibility to advocate in his or her area of expertise. It is imperative that nurse and healthcare leaders address these disparities at not only on a national level but also on a global one Huber, 2018).

For instance, on a national level, one example is the undocumented population. Health disparities often occur due to lack of health insurance and access to healthcare (Huber, 2018). In the United States alone there are more than 12 million individuals without access to healthcare, many with chronic or long-term ailments (Sorrell, 2017). Lack of decent health coverage is an ethical issue because adequate healthcare is a basic human right and need (Sorrell, 2017). Healthcare should be fair, safe and impartial, and should improve the health of the population (Huber, 2018). One of the drawbacks is that healthcare and nursing leaders have lack of guidance concerning the authorization and delivery of healthcare to this population (Sorrell, 2017).

How should the nursing leader advocate for these immigrants?  Nurse leaders and other healthcare providers need to publicly recommend and support policies that provide lasting and impartial health provisions for these individuals (Sorrell, 2017).  Nurse leaders need to join with lawmakers and organizations such as the Center on Health Equity Action for System Transformation or the Health Equity Task Force to help develop a health equity centered system (Families USA, 2019).

The wife and daughter need teaching about his antiepileptic medication. What teaching should be included?

An older client was recently discharged from the hospital for evaluation of seizure activity. His history reveals that he has late-stage Alzheimer’s disease, Parkinson’s disease, hypertension, and type II diabetes mellitus, which is controlled by diet. He lives at home, where his wife and daughter take care of him. His discharge medications include phenytoin (Dilantin), 100 mg BID; hydrochlorothiazide (HydroDIURIL), 50 mg QD; levodopa (Sinemet), 25/100 TID; and haloperidol (Haldol), 1 mg before bed. The client has been referred for home care nursing follow-up.

Questions:

  1. On the initial home visit by the nurse, what assessments should be made?
  2. The wife and daughter need teaching about his antiepileptic medication. What teaching should be included?
  3. During the initial home visit, the client experiences a generalized seizure. What action should the nurse take?

Provide a meaningful response to at least two of your peers’ posts.

Provide a meaningful response to at least two of your peers’ posts.  Also include your discussion some citations to support your statements. It is good to have opinions, but always best to have some evidence-based information to back up your points.

First peer:The community nurse’s role consists of many things when assisting patients, families and the community as a whole. They help provide screenings for cancers such as mammograms, STI testing, and treatments measures for communicable diseases and management for some long term diseases such as diabetes. Research, screenings, treatment measures and educating the patient, families and the community the nurse serves accounts for the nurses’ responsibilities. Research may consist of figuring out why certain diseases are affecting the communities and to what extent and how to treat. Research also includes networking and finding the funding needed to support research and help pay for the cost to provide screenings, treatments as well as resources such as flyers, brochures, and videos to help educate.

First peer: The community nurse’s role consists of many things when assisting patients, families and the community as a whole. They help provide screenings for cancers such as mammograms, STI testing, and treatments measures for communicable diseases and management for some long term diseases such as diabetes. Research, screenings, treatment measures and educating the patient, families and the community the nurse serves accounts for the nurses’ responsibilities. Research may consist of figuring out why certain diseases are affecting the communities and to what extent and how to treat. Research also includes networking and finding the funding needed to support research and help pay for the cost to provide screenings, treatments as well as resources such as flyers, brochures, and videos to help educate.

Second peer: Community health nurses focus on long- and short-term care for disease prevention, community health nurses focus on presenting materials in a clear and understandable format. Public health nurses advocate on the local, state, and federal level to provide better access to health care, protect funding for public health programs, and reduce or eliminate health disparities. Community nurses cross-cultural, language, and literacy boundaries to shape the health and well-being of children and adults.  As a community health nurse is an excellent way to combine a love of nursing and a love of your community.  Community nurses often work in community health centers offering treatment and advice to members of the community. Community health nurses can also help provide the necessary care in communities that lack easily accessible healthcare.   Barriers or challenges the nurse would need to overcome to achieve these goals is Higher poverty rates, which can make it difficult for participants to pay for services or program. Cultural and social norms surrounding health behaviors, Low health literacy levels and incomplete perceptions of health, Limited affordable, reliable, and public transportation options.

Write several broad instructional goals for the educational experience.

Course Project—Part 2

Instructional Unit: Goals, Objectives, and the Teaching Plans

This week you will submit your instructional unit, which should include three lesson plans: one focused on patient education, one on family education, and one on staff development. Remember, your plans should demonstrate a logical approach to teaching, communicate what is to be taught and how, and outline how objectives are to be evaluated. Click here for a refresher on what each final lesson should include.

Building on the work that you started in Week 2, for all three groups of learners:

  • Write several broad instructional goals for the educational experience.
  • Write several behavioral objectives based on Bloom’s taxonomy.
  • Describe the lesson content.
  • Provide a sequence for teaching activities.
  • Describe instructional methods.
  • Indicate time allotted for each activity.
  • Identify and describe the instructional resources (materials, tools, etc.) and technology to be used.
  • Describe how the learning will be evaluated.

On a separate references page, cite all sources using APA format.

Submission Details:

  • Submit your response in a 6- to 7-page paper to the Submissions Area by the due date assigned. Be sure to use appropriate APA formattingLesson Components

    At a minimum, each final lesson must contain the following components: Introduction:

     Provide the title of the lesson.  Identify and describe the learners.  Include Learner Assessment: educational level, developmental level, readiness to learn, etc.  Describe the educational setting: (staff development, patient education, family education,

    etc.). Purpose and rationale for the lesson(s):

     Provide a rationale for selecting this lesson.  Describe the philosophical or theoretical basis for teaching approaches used in the lesson.

    Statement of goals and objectives:

     Write several broad instructional goals for the educational experience.  Write several behavioral objectives based on Bloom’s taxonomy.

    Instructional methods and evaluation of learning—For each objective:

     Describe the lesson content.  Provide a sequence for teaching activities.  Describe instructional strategies.  Indicate time allotted for each activity.  Describe the instructional resources (materials, tools, etc.) to be used.  Describe how the learning will be evaluated.

Do you see the importance of writing a resignation letter and tell why you do or do not think it is important? 

Let us think a bit further out into your career and say you have been working at your current job position a year. You have a yearning to apply to another unit in the same facility or you are moving to another state, or you just want to try another nursing career field. How do you give up this current job eloquently, without hard feelings or “burning bridges”? Do you see the importance of writing a resignation letter and tell why you do or do not think it is important?

Analyze how the person functioned within the structural frame.

When you have finished reading the biography on your selected leader, and with the help of your guidance outline (Submitted in Week 9), complete your paper that analyzes the person through the four frames of leadership: structural, human resources, political, and symbolic. Address how personality, leadership style, influence, and motivation factored into the successes and failures of the person. The structure of your paper should be as follows:

  1. Introduction: Who you selected and why (remember this is not a first person paper).
  2. Background of the person.
  3. Explain how do personality, leadership style, influence, and motivation factored into the persons success or failure (these points may not be explicitly addressed in your biography, but you should have enough information based on our coursework to make some logical determinations about these aspects of your selected person).
  4. Analyze how the person functioned within the structural frame.
  5. Analyze how the person functioned within the human resource frame.
  6. Analyze how the person functioned within the political frame.
  7. Analyze how the person functioned within the symbolic frame.
  8. Explain how the four frame interacted, impacted, and/or influenced the person’s perspective.
  9. Conclusion

Remember that your final paper submission is not a question and answer document. You are to analyze the information you find, organize it, and present it in a paper that is 12-15 pages in length. Your paper should be written at the college level, adhere to APA guidelines, and include a reference list. Any sources used must be cited properly, including the biography, using in-text citations and a references page.

Discuss why EBP is an essential component of the practice of a BSN-prepared RN

Discuss why EBP is an essential component of the practice of a BSN-prepared RN. Identify two ways in which you will continue to integrate evidence into your practice and encourage it within your work environment. What obstacles could challenge this plan, and what steps will you take to minimize their impact?

Post a description of the improvement initiative(s) currently in place at your Health Service Organization (HSO)  or an HSO with which you are familiar.

Consider the following scenario:

Rene is a health care administrator who works for an Health Service Organization (HSO) in a large urban area. Because she was previously a health care provider, Rene is sensitive to the need for health care delivery that also promotes patient safety. As part of the responsibilities in working within the HSO, the board of directors has asked Rene about the needs of the HSO for fostering improvement initiatives. To that end, Rene suggested initiatives aimed at increasing patient safety in light of a recent report citing poor patient safety across all departments in the HSO. The board agreed and has made Rene responsible for developing a new patient safety initiative. Rene quickly communicated with several department leads to determine potential courses of action to improve patient safety within the HSO. A meeting to hear from each respective department with contributions on how to implement a new patient initiative is slated for next week. Several department heads and the board of the HSO have been impressed with the commitment that Rene has demonstrated thus far.

How are improvement initiatives put into place within HSOs? How might leadership and the approaches used by leaders contribute to the implementation of improvement initiatives in practice? As a current or future health care administrator, you will likely encounter improvement initiatives aimed to increase the efficiency and effectiveness of health care delivery.

For this Discussion, reflect on the scenario presented and consider the role of leadership for improvement initiatives in HSOs. Then, think about improvement initiatives currently in place at your HSO or an HSO with which you are familiar.

Post a description of the improvement initiative(s) currently in place at your Health Service Organization (HSO)  or an HSO with which you are familiar. Explain how leadership is related to the particular improvement initiative selected. Then, explain the levels of the integrated system of the HSO, and suggest two approaches that leadership might use to work within these levels to execute the improvement initiative you selected. Be specific and provide examples.

What were the consequences of a failure to report?

View the scenario called “Critical Decision Making for Providers” found in the Allied Health Community media (https://lc.gcumedia.com/hlt307v/allied-health-community/home.html).

In a 750‐1,200 word paper, describe the scenario involving Mike, the lab technician, and answer the following questions:

  1. What were the consequences of a failure to report?
  2. What impact did his decision have on patient safety, on the risk for litigation, on the organization’s quality metrics, and on the workload of other hospital departments?
  3. As Mike’s manager, what will you do to address the issue with him and ensure other staff members do not repeat the same mistakes?

A minimum of three academic references from credible sources are required for this assignment.

Prepare this assignment according to the APA guidelines found in the APA Style Guide, located in the Student Success Center. An abstract is not required.

This assignment uses a grading rubric. Instructors will be using the rubric to grade the assignment; therefore, students should review the rubric prior to beginning the assignment to become familiar with the assignment criteria and expectations for successful completion of the assignment.

You are required to submit this assignment to LopesWrite. Refer to the directions in the Student Success Center. Only Word documents can be submitted to LopesWrite.

For education purposes during the mandatory class that all potential preceptor’s need to attend before being qualified to precept another colleague.

Life and the health care system are dominated by technology, and most of the impact that technology has on health care is positive. However, according to Taylor (2013) there are also risks involved with the extensive use of technology in healthcare. For example, communication errors and breach in confidentiality can quickly happen through the widespread use of technology (Taylor, 2013).

Technology is an integral supportive and integrative part of my project implementation process. The project focus is development and implementation of a formal unit-based preceptor program. Following are some ways that computer technology will be used for my change project initiative:

  • For education purposes during the mandatory class that all potential preceptor’s need to attend before being qualified to precept another colleague.
  • To develop a structured unit-based plan that all preceptors and preceptees will follow during the orientation phase.
  • To measure preceptor expectations and performance during the orientation of a preceptee.
  • It will be used as an evaluation tool to document and measure preceptee goals, performance, and satisfaction.
  • It will be used as a tool to track and measure patient outcomes, nurse retention rates, nurse competency and skills, and job satisfaction which are evidence-based research outcomes that are expected to increase with a structured formal unit-based preceptor program.

Since computer technology is already an integral part of the work process on the unit that this project initiative will be implemented, I see no barriers that would prevent the use of it for the focused project change. Many computer technology advances in health care will continue to be used to provide safe environments for patients and their families (Taylor, 2013).

Reference

Taylor, K. (2013). Technology in general practice—is it risky business? Practice Nurse, 

43(5), 10-11. Retrieved from https://search-ebscohost-

com.lopes.idm.oclc.org/login.aspx?direct=true&db=a9h&AN=87833894&site=eds-

What is a safe treatment plan for a patient with Atrial Fibrillation?

Please provide proper citation with page numbers of the material you are referencing in your Main Post.
Please write 1 paragraph for each of the following (total 3 paragraphs) and each paragraph shall be no less than 3 sentences and no more than 6 sentences.

  1. Discuss one of the following: STEMI, Non-STEMI; congestive heart failure; murmurs; ventricular tachycardia; asystole
  2. What is a safe treatment plan for a patient with Atrial Fibrillation?
  3. What is a safe treatment plan to reduce a patient’s bilateral lower extremities pitting edema?

What is a safe treatment plan for a patient with Atrial Fibrillation?

Please provide proper citation with page numbers of the material you are referencing in your Main Post.
Please write 1 paragraph for each of the following (total 3 paragraphs) and each paragraph shall be no less than 3 sentences and no more than 6 sentences.

  1. Discuss one of the following: STEMI, Non-STEMI; congestive heart failure; murmurs; ventricular tachycardia; asystole
  2. What is a safe treatment plan for a patient with Atrial Fibrillation?
  3. What is a safe treatment plan to reduce a patient’s bilateral lower extremities pitting edema?

Why is their support necessary to the success of your project, and how you will go about securing that support?

Stakeholder support is necessary for a successful change proposal project implementation. Consider your internal stakeholders, such as the facility, unit or health care setting where the change process is situated, and your external stakeholders, like an individual or group outside the health care setting. Why is their support necessary to the success of your project, and how you will go about securing that support?  (Topic is infection control, and mentorship will be in a NY elementary school).

Please include in-text citations, and references must be no older than 5 years.

Post the audience and learning need that you have selected.

  • Select an audience of learners (nursing students, nursing staff, or patients) that you are interested in teaching.
  • Reflect on the diverse learning needs of this specific audience and select one to further investigate for your Discussion*.
  • Review the article, Writing Learning Objectives that Help You Teach and Students Learn (Part 1), and the links to the Lesson Plan Tutorials, which are located in this week’s Learning Resources. Reflect on the examples and non-examples of action verbs. Then, consider the action verbs that you might select to address the identified learning need.
  • Review the Crafting Learning Objectives document, which is located in this week’s Learning Resources. Then, craft at least two learning objectives—relevant to the learning need that you identified, which follow the format required of the Nursing Education specialization.
  • Consider learning activities that might align to these learning objectives. For example, if the learning objective is to demonstrate the ability to accurately take a patient’s blood pressure, the activity should involve students practicing the process of blood pressure readings; if the learning objective is to compare learning theories, the activity may include a small group discussion.
  • Consider how each activity could be used to meaningfully assess student, staff, or patient learning.
  • Keeping the audience and learning need in mind, create a description of a learning activity that aligns to the objectives.

* Select an audience and learning need about which you are genuinely interested. You will keep this focus (audience, learning need, learning objectives) and expand on it over the next three Discussions.

By Day 3

Post the audience and learning need that you have selected. Describe the learning activity that you might use to address this need and identify at least two appropriate learning objectives. Explain how each objective aligns to the learning activity, as well as how each objective can be used to meaningfully assess student, staff, or patient learning. Justify your response with references to this week’s Learning Resources and current literature.

What are some facilitating and inhibiting factors with regard to leadership and this issue

As you engage in your practicum experience, develop your journal entry, and prepare for the Discussion, keep the following questions in mind:

  • What is the role of leadership in the change process—specifically as it relates to your practicum setting and the change you have identified? What are some facilitating and inhibiting factors with regard to leadership and this issue? Refer to leadership and change theory.
  • What opportunities and challenges could arise based on your personal leadership style as well as your role within your practicum setting? Consider the results of the DiSC, which you completed in NURS 8300.

How can nurses facilitate patient and family focused care?

As nurses, we want to provide the best care possible, but what does that look like? How do we know that what we are doing is the best practice for our patients? Is the care we provide focused on hospital regulations, the routines of the nurses, the conveniences of the doctors, or the needs of the patient and his or her family?

In the 1950’s and 1960’s, patient care was so hospital centric that fathers were not allowed in the delivery room, patients were routinely subjected to unnecessary painful and often dangerous procedures, and mothers and babies were routinely separated after birth. We need to re-think the way we provide care, based on evidence, focused on the patient and driven by patient outcomes. There are some forward thinking individuals who have begun to lead the way.

Do some research on the practice of patient-centered care. Think about how you would want to be cared for if you were the patient, or if your loved one were the patient. What is meant by the terms mother friendly and baby friendly? How can nurses facilitate patient and family focused care?

Introduce yourself; state the issue that you will address in the letter that is causing the problem. Include bill information if applicable. 

Websites for references

American Nurses Association (ANA) Policy and Advocacy. Here you can find state legislative issues the ANA is promoting across the country. http://www.nursingworld.org/MainMenuCategories/Policy-Advocacy

Florida Nurses Association-Nursing, Advocacy, Engagement http://cqrcengage.com/fln/home?0

Florida Center for Nursing (FCN) and the Action Coalition (FL-AC) https://www.flcenterfornursing.org/FL-ActionCoalition.aspx

Follow the steps outlined below to write a letter that will be emailed to your Congressperson or Senator to express your view on an issue in nursing that is negatively impacting or will negatively impact health care. The letter/email should reflect professionalism and be proof-read. It is recommended that you write your letter on a Word document which you will then copy and paste into the email.

Address the following in about 4 paragraphs to include:

1. Introduce yourself; state the issue that you will address in the letter that is causing the problem. Include bill information if applicable.

2. Your opinion on this issue. Why is your issue important for you as a registered nurse? Research the issue to support your argument. Include current socioeconomic, political, or ethical issues surrounding the problem as well as the implications of the problem for patients, nursing or nursing practice.

3. Restate your purpose for writing the letter. Ask the Congressperson/Senator to help to take action to address the issue.

4. Ensure that you thank the Congressperson/Senator (MY STATE SENATOR IS MARCO RUBIO

Identify and discuss at least two potential ethical issues that could be of concern for nurses with telehealth delivered care.

1 Identify and discuss at least two potential ethical issues that could be of concern for nurses with telehealth delivered care.

Submission Instructions:

  • Your initial post should be at least 500 words, formatted and cited in current APA style with support from at least 2 academic sources.
  • Identify and demonstrate a sophisticated understanding of the issues, problems, and concept
  • Effectively uses the literature and other resources to inform their work. Exceptional use of citations and extended referencing. High level of APA precision and free of grammar and spelling errors.

What sorts of devices, rhetorical, language, or other are employed to either enhance or distract the audience from the message?

For your written assignment, find three advertisements. These can be  videos, radio ads, print ads, interactive media, or any other sort of  media used for advertising purposes. For each advertisement, provide a ½ page analysis that answers the following questions:

  • Summarize the advertisement, the apparent demographic it is aimed at, and the product it is pitching.
  • What are the arguments being made? Are they explicit or only implied?
  • Are the arguments being forwarded cogent? Why or why not?
  • What sorts of devices, rhetorical, language, or other are employed to either enhance or distract the audience from the message?

Reflect on the feedback you received from your colleagues on your Discussion post for the national healthcare issue/stressor you selected. 

The Quadruple Aim provides broad categories of goals to pursue to maintain and improve healthcare. Within each goal are many issues that, if addressed successfully, may have a positive impact on outcomes. For example, healthcare leaders are being tasked to shift from an emphasis on disease management often provided in an acute care setting to health promotion and disease prevention delivered in primary care settings. Efforts in this area can have significant positive impacts by reducing the need for primary healthcare and by reducing the stress on the healthcare system.

Changes in the industry only serve to stress what has always been true; namely, that the healthcare field has always faced significant challenges, and that goals to improve healthcare will always involve multiple stakeholders. This should not seem surprising given the circumstances. Indeed, when a growing population needs care, there are factors involved such as the demands of providing that care and the rising costs associated with healthcare. Generally, it is not surprising that the field of healthcare is an industry facing multifaceted issues that evolve over time.

In this module’s Discussion, you reviewed some healthcare issues/stressors and selected one for further review. For this Assignment, you will consider in more detail the healthcare issue/stressor you selected. You will also review research that addresses the issue/stressor and write a white paper to your organization’s leadership that addresses the issue/stressor you selected.

To Prepare:

  • Review the national healthcare issues/stressors presented in the Resources and reflect on the national healthcare issue/stressor you selected for study.
  • Reflect on the feedback you received from your colleagues on your Discussion post for the national healthcare issue/stressor you selected.
  • Identify and review two additional scholarly resources (not included in the Resources for this module) that focus on change strategies implemented by healthcare organizations to address your selected national healthcare issue/stressor.

The Assignment (3-4 Pages):

Analysis of a Pertinent Healthcare Issue

Develop a 3- to 4-page paper, written to your organization’s leadership team, addressing your selected national healthcare issue/stressor and how it is impacting your work setting. Be sure to address the following:

  • Describe the national healthcare issue/stressor you selected and its impact on your organization. Use organizational data to quantify the impact (if necessary, seek assistance from leadership or appropriate stakeholders in your organization).
  • Provide a brief summary of the two articles you reviewed from outside resources on the national healthcare issue/stressor. Explain how the healthcare issue/stressor is being addressed in other organizations.
  • Summarize the strategies used to address the organizational impact of national healthcare issues/stressors presented in the scholarly resources you selected. Explain how they may impact your organization both positively and negatively. Be specific and provide examples.

Provide at least Five Targeted Questions you would ask the patient to build his or her health history and to assess his or her health risks.

Provide at least Five Targeted Questions you would ask the patient to build his or her health history and to assess his or her health risks.

“CASE STUDY #1”

JC, an at-risk 86-year-old Asian male is physically and financially dependent on his daughter, a single mother who has little time or money for her father’s health needs. He has ahx of hypertension (HTN), gastroesophageal reflux disease (GERD), b12 deficiency, and chronic prostatitis. He currently takes Lisinopril 10mg QD, Prilosec 20mg QD, B12 injections monthly, and Cipro 100mg QD. He comes to you for an annual exam and states “I came for my annual physical exam, but do not want to be a burden to my daughter.”

Note: REMEMBER AT LEAST THREE REFERENCES AND APA FORMAT

Explain the stepwise approach to asthma treatment and management for your patient.

Create a 5- to 6-slide PowerPoint presentation that can be used in a staff development meeting on presenting different approaches for implementing the stepwise approach for asthma treatment. Be sure to address the following:

  • Describe long-term control and quick relief treatment options for the asthma patient from your practice as well as the impact these drugs might have on your patient.
  • Explain the stepwise approach to asthma treatment and management for your patient.
  • Explain how stepwise management assists health care providers and patients in gaining and maintaining control of the disease. Be specific.

Based on the results of the SWOT analysis, what should Barbara recommend as an overall strategy?

Now that Barbara has an understanding of the factors that influence clinic performance, it is time to set priorities and develop a preliminary strategy. A clear strategy helps to focus energy and resources, establish organizational direction, and strengthen operations where there are opportunities for improvement.

For this project assignment on UCCO, complete a 2-page report to describe a high-level strategy with associated action items for, to include the following concepts:

  • Based on the results of the SWOT analysis, what should Barbara recommend as an overall strategy?
  • How will the selection of the chosen EHR system contribute to the strategy? Further explain why it was the best choice.
  • On what basis should she develop actions items? What should the action items be, as they directly relate to the strategy?
  • What should the recommended outcomes and/ or long-term goals be, based on the action items?
  •  APA formatted reference page

Write the incident down here by writing word for word, as best as you can remember, what the patient said or did that prompted the question

Think about a time when you were with a distressed patient and you found yourself asking, “What do I say?” Write the incident down here by writing word for word, as best as you can remember, what the patient said or did that prompted the question. Be sure to protect the anonymity of the patient by using a pseudonym.

Apply leadership concepts, skills, and decision making in the provision of high-quality nursing care, healthcare team management, and the oversight and accountability for care delivery in a variety of settings.

Purpose

This week’s graded discussion topic relates to the following Course Outcome (CO).

  • CO 1: Apply leadership concepts, skills, and decision making in the provision of high-quality nursing care, healthcare team management, and the oversight and accountability for care delivery in a variety of settings. (PO 2)

Discussion

Healthcare reform is a term that is ever present in our practice settings and will not disappear any time soon.

Review the article below and answer the following questions.

Buerhaus, P. I., Skinner, L. E., Auerbach, D. I., & Staiger, D. O. (2017). Four challenges facing the nursing workforce in the United States. Journal of Nursing Regulation, 8(2), 40-46. . doi:10.1016/S2155-8256(17)30097-https://chamberlainuniversity.idm.oclc.org/login?url=https://search.ebscohost.com/login.aspx?direct=true&db=edselp&AN=S2155825617300972&site=eds-live&scope=site (Links to an extern

 

  1. Select one of the four challenges and expand on how your workforce has responded to the change.
  2. How have these challenges affected the access to care for your patients?
  3. What can you do to decrease the effect on your patients? Provide a specific example.

Defend how your area of interest is of importance to the MSN program specialty track you have selected—scholarly references are required.

300 words, 1 scholarly reference, no later than 5 years.

State your area of evidence-based practice (EBP) interest consistent with your specialty track.

State your MSN program specialty track and defend how your selected area of EBP interest is consistent with your selected MSN program track.

Defend how your area of interest is of importance to the MSN program specialty track you have selected—scholarly references are required.

Identify an initial PICOT question that reflects your area of interest for the evidence-based practice proposal.

Review the national healthcare issues/stressors presented in the Resources and reflect on the national healthcare issue/stressor you selected for study.

The Quadruple Aim provides broad categories of goals to pursue to maintain and improve healthcare. Within each goal are many issues that, if addressed successfully, may have a positive impact on outcomes. For example, healthcare leaders are being tasked to shift from an emphasis on disease management often provided in an acute care setting to health promotion and disease prevention delivered in primary care settings. Efforts in this area can have significant positive impacts by reducing the need for primary healthcare and by reducing the stress on the healthcare system.

Changes in the industry only serve to stress what has always been true; namely, that the healthcare field has always faced significant challenges, and that goals to improve healthcare will always involve multiple stakeholders. This should not seem surprising given the circumstances. Indeed, when a growing population needs care, there are factors involved such as the demands of providing that care and the rising costs associated with healthcare. Generally, it is not surprising that the field of healthcare is an industry facing multifaceted issues that evolve over time.

In this module’s Discussion, you reviewed some healthcare issues/stressors and selected one for further review. For this Assignment, you will consider in more detail the healthcare issue/stressor you selected. You will also review research that addresses the issue/stressor and write a white paper to your organization’s leadership that addresses the issue/stressor you selected.

To Prepare:

  • Review the national healthcare issues/stressors presented in the Resources and reflect on the national healthcare issue/stressor you selected for study.
  • Reflect on the feedback you received from your colleagues on your Discussion post for the national healthcare issue/stressor you selected.
  • Identify and review two additional scholarly resources (not included in the Resources for this module) that focus on change strategies implemented by healthcare organizations to address your selected national healthcare issue/stressor.

The Assignment (3-4 Pages):

Analysis of a Pertinent Healthcare Issue

Develop a 3- to 4-page paper, written to your organization’s leadership team, addressing your selected national healthcare issue/stressor and how it is impacting your work setting. Be sure to address the following:

  • Describe the national healthcare issue/stressor you selected and its impact on your organization. Use organizational data to quantify the impact (if necessary, seek assistance from leadership or appropriate stakeholders in your organization).
  • Provide a brief summary of the two articles you reviewed from outside resources on the national healthcare issue/stressor. Explain how the healthcare issue/stressor is being addressed in other organizations.
  • Summarize the strategies used to address the organizational impact of national healthcare issues/stressors presented in the scholarly resources you selected. Explain how they may impact your organization both positively and negatively. Be specific and provide examples.

From the previous discussion:

The healthcare environment is significantly growing, which explains the reason for the existing significant issues. Improving the quality of healthcare requires a strong emphasis on critical aspects such as leadership, cost of care, skills and service delivery, technology, as well as patient management practices. A significant health care issue currently has been technological disruption. Substantial changes in the healthcare industry that are being undertaken are dependent on technology. This provides a unique platform where it is more accessible to attain better health outcomes across different settings.

Quality has been a critical measure in the delivery of quality healthcare outcomes. The integration of different results has aimed at providing a unique system where it would be easier to improve healthcare outcomes. The ability to incorporate technology in monitoring patient development offers a strong emphasis on essential processes which help maintain a unique level of engagement within a healthcare setting. Within my healthcare organization, patient information has been stored using healthcare information management systems compared to previous documentation of patient information in healthcare files (Engelhardt, 2017).

Management of healthcare information has become a critical factor which provides a strong consideration on critical processes which help create a reliable system. The inclusion of health information management systems has integrated vital processes which help identify important concepts which help manage patient information. Storage of patient details provides a strong consideration on specific aspects which help improve the level of commitment among healthcare providers (Sultan, 2015).  The health information management system that has been put in place identify specific measures which focus on ensuring that there is quick information retrieval of patient information.  Accessing patient information within a healthcare context requires authorization from a patient or when reviewing the patient situation.

The impact of technology disruption in my workplace

Integration of technology in healthcare is aimed at controlling healthcare errors. It is a crucial aspect to consider primarily based on previous errors in healthcare delivery.  The use of technology is aimed at improving efficiency in the delivery of healthcare. In my workplace, management of patient health information is a significant improvement to the storage of health information which has been a considerable challenge, especially retrieval of patient files. The use of technology has made it possible to access patient information from a cloud-based database quickly (Schwamm, 2014).

Conclusion

The use of technology has also created challenges, especially concerning the common cyber threats. The fact that we deal with sensitive patient information, they are at a higher risk of falling victims to intruders who target such information.   The present risks are significant. Thus, integration of vital information security risks will help in controlling the current risks through the development of robust techniques. The techniques are integrated into the healthcare industry to protect patient information. Thus, management of these risks has been a significant aspect to consider in ensuring that there is a better level of implementation of healthcare outcomes. Thus, technology disruption is having both positive and negative influence in healthcare.

References

Engelhardt, M. A. (2017). Hitching healthcare to the chain: An introduction to blockchain technology in the healthcare sector. Technology Innovation Management Review, 7(10).

Schwamm, L. H. (2014). Telehealth: seven strategies to successfully implement disruptive technology and transform health care. Health Affairs, 33(2), 200-206.

Sultan, N. (2015). Reflective thoughts on the potential and challenges of wearable technology for healthcare provision and medical education. International Journal of Information Management, 35(5), 521-526.

or

Increased workload on nurses is one of the stressors facing nurses and is a significant problem all over the world. It includes multiple factors such as work environment, condition of the client or consumer, characteristics of the providers, nursing interventions, to name a few.  Contributing further to the already heavy nursing workload are an inadequate supply of nurses, decreased patient length of stay, complex patients, increased demand for nurses, and increased overtime had added to already demanding work of nurses and eventual nursing burnout. As a result, the nurse-patient ratio, which has been used commonly in measuring the workload of the nursing workforce, has been increasing (Vaidya, 2018).

reference:

Vaidya, A. (2017, September 21). Increasing nurse’s workload by 1 patient ups odds of patient

death by 7%. Clinical Leadership and Infection Control. Retrieved from

https://www.beckershospitalreview.com/quality/increasing-nurse-s-workload-by-1-patient-ups-odds-of-patient-death-by-

7.html

Looking Ahead

The paper you develop in Module 1 will be revisited and revised in Module 2. Review the Assignment instructions for Module 2 to prepare for your revised paper.

rubic:

 

Develop a 3- to 4-page paper, written to your organization’s leadership team, addressing the selected national healthcare issue/stressor and how it is impacting your work setting. Be sure to address the following:

·   Describe the national healthcare issue/stressor you selected and its impact on your organization. Use organizational data to quantify the impact (if necessary, seek assistance from leadership or appropriate stakeholders in your organization).–Levels of Achievement:Excellent 23 (23%) – 25 (25%)
The response accurately and thoroughly describes in detail the national healthcare issue/stressor selected and its impact on an organization.

The response includes accurate, clear, and detailed data to quantify the impact of the national healthcare issue/stressor selected.
Good 20 (20%) – 22 (22%)
The response describes the national healthcare issue/stressor selected and its impact on an organization.

The response includes accurate data to quantify the impact of the national healthcare issue/stressor selected.
Fair 18 (18%) – 19 (19%)
The response describes the national healthcare issue/stressor selected and its impact on an organization that is vague or inaccurate.

The response includes vague or inaccurate data to quantify the impact of the national healthcare issue/stressor selected.
Poor 0 (0%) – 17 (17%)
The response describes the national healthcare issue/stressor selected and its impact on an organization that is vague and inaccurate, or is missing.

The response includes vague and inaccurate data to quantify the impact of the national healthcare issue/stressor selected, or is missing.Feedback:

·   Provide a brief summary of the two articles you reviewed from outside resources, on the national healthcare issue/stressor and explain how the healthcare issue/stressor is being addressed in other organizations.–Levels of Achievement:Excellent 27 (27%) – 30 (30%)
A complete, detailed, and specific synthesis of two outside resources reviewed on the national healthcare issue/stressor selected is provided. The response fully integrates at least 2 outside resources and 2 or 3 course-specific resources that fully support the summary provided.

The response accurately and thoroughly explains in detail how the healthcare issue/stressor is being addressed in other organizations.
Good 24 (24%) – 26 (26%)
An accurate synthesis of at least one outside resource reviewed on the national healthcare issue/stressor selected is provided. The response integrates at least 1 outside resource and 2 or 3 course-specific resources that may support the summary provided.

The response fails to integrate any resources to support the summary provided.Feedback:

·   Summarize the strategies used to address the organizational impact of national healthcare issues/stressors presented in the scholarly resources you selected and explain how they may impact your organization both positively and negatively. Be specific and provide examples.–Levels of Achievement:Excellent 27 (27%) – 30 (30%)
A complete, detailed, and accurate summary of the strategies used to address the organizational impact of the national healthcare issue/stressor is provided.

 

The response explains how the strategies may impact an organization both positively and negatively that is vague and inaccurate, or is missing. Does not include any examples.Feedback:

Written Expression and Formatting – Paragraph Development and Organization:

Paragraphs make clear points that support well-developed ideas, flow logically, and demonstrate continuity of ideas. Sentences are carefully focused—neither long and rambling nor short and lacking substance. A clear and comprehensive purpose statement and introduction is provided which delineates all required criteria.–Levels of Achievement:Excellent 5 (5%) – 5 (5%)
Paragraphs and sentences follow writing standards for flow, continuity, and clarity.

Written Expression and Formatting – The paper follows correct APA format for title page, headings, font, spacing, margins, indentations, page numbers, parenthetical/in-text citations, and reference list.–Levels of Achievement:Excellent 5 (5%) – 5 (5%)
Uses correct APA format with no errors.

Developing a planning strategy for your persuasive speech

Developing a planning strategy for your persuasive speech

Develop a planning strategy for your persuasive speech based on the following five criteria for an effective speech:

  1. need for credibility
  2. need for intellectual stimulation
  3. need for creativity
  4. need for relevance

Use these criteria to answer the following questions about your own speech:

  • How would you demonstrate that you are a credible speaker on this subject?
  • How would you keep the audience thinking and learning about your topic?
  • How would you make the speech creative?
  • How would you demonstrate that this topic is relevant to the audience’s needs and interests?

Be specific in your answers. For instance, don’t just say you are a credible speaker because you have done a lot of research. You need to give us what sources you are using and how you have determined they are credible. For creativity, don’t just say you will use a PowerPoint. You need to tell us what will be on each slide.

Before the end of the week, begin commenting on at least 2 of your peers’ responses. You can ask technical questions or respond generally to the overall experience. Be objective, clear, and concise. Always use constructive language, even in criticism, to work toward the goal of positive progress. Submit your responses to the appropriate topic in this

How do you believe they would be ordered in the context of the Christian biblical narrative?

Principlism, especially in the context of bioethics in the United States, has often been critiqued for raising the principle of autonomy to the highest place, such that it trumps all other principles or values. How would you rank the importance of each of the four principles? How do you believe they would be ordered in the context of the Christian biblical narrative? Refer to the topic overview and other topic study materials in your response.

Compare the pathophysiology of chronic venous insufficiency and deep venous thrombosis.

Write a 2- to 3-page paper that addresses the following:

  • Compare the pathophysiology of chronic venous insufficiency and deep venous thrombosis. Describe how venous thrombosis is different from arterial thrombosis.
  • Explain how the patient factor you selected might impact the pathophysiology of CVI and DVT. Describe how you would diagnose and prescribe treatment of these disorders for a patient based on the factor you selected.
  • Construct two mind maps—one for chronic venous insufficiency and one for deep venous thrombosis. Include the epidemiology, pathophysiology, and clinical presentation, as well as the diagnosis and treatment you explained in your paper.

Explain your diagnosis for the patient, including your rationale for the diagnosis.

Write a 1-page paper that addresses the following:

  • Explain your diagnosis for the patient, including your rationale for the diagnosis.
  • Describe an appropriate drug therapy plan based on the patient’s history, diagnosis, and drugs currently prescribed.
  • Justify why you would recommend this drug therapy plan for this patient. Be specific and provide examples.

describe the teaching experience and discuss your observations.

Note: This is an individual assignment. In 1,500-2,000 words, describe the teaching experience and discuss your observations. The written portion of this assignment should include:

  1. Summary of teaching plan
  2. Epidemiological rationale for topic
  3. Evaluation of teaching experience
  4. Community response to teaching
  5. Areas of strengths and areas of improvement

Prepare this assignment according to the APA guidelines found in the APA Style Guide

Explain why a developmental assessment of children and adolescents is important.

Discussion: Working With Children and Adolescents Versus Adults

Infant, childhood, and adolescent development are a continual interplay between nature (genetic or biologic predisposition) and nurture (environmental experiences). The nature/nurture continuum and debate will always be a part of your career as a PMHNP.  Knowing common developmental milestone is important in the role as a child provider. Not only is it essential to the diagnostic process, but it is also important to the interdisciplinary interactions with other mental health professionals. The study of normal developmental processes, however, is only one tool that allows the mental health professional to understand the child being evaluated. There are many different assessment instruments and interviewing techniques that PMHNPs can have in their toolkit when working with children and adolescents.

In this Discussion, you examine the differences in assessing and treating children and adolescents versus adults. You take into consideration your own clinical experiences, as well as your experiences in your clinical rotation, and the information from the readings thus far.

Post your answers to the following:

  • Explain why a developmental assessment of children and adolescents is important.
  • Describe two assessment instruments and explain why they are used for children and adolescents but not adults.
  • Describe two treatment options for children and adolescents that may not be used when treating adults.
  • Explain the role parents play in assessment and treatment.

Assignment: Practicum Journal Entry: Analyzing an Ethical Decision

Assignment: Practicum Journal Entry: Analyzing an Ethical Decision

In your role as a PMHNP, you will encounter several situations that will require your ability to make sound judgments and practice decisions for the safety and well-being of individuals, families, and communities. There may not be a clear-cut answer of how to address the issue, but your ethical decision making must be based on evidenced-based practice and what is good, right, and beneficial for patients. You will encounter patients who do not hold your values, but you must remain professional and unbiased in the care you provide to all patients regardless of their socio-demographic and ethnic/racial background. You must be prepared to critically analyze ethical situations and develop an appropriate plan of action. For this Assignment, you review the literature and discover the various ethical dilemmas PMHNPs encounter and how these issues are typically addressed in your state.

To prepare:

  • Review literature for moral/ethical issues encountered by a PMHNP.
  • Select one of the articles you found that was published within the last 5 years to use as a focus for this assignment.

Write a 2-page paper in which you do the following:

  • Summarize the moral/ethical issue in the article (no more than 1 paragraph).
  • Describe the moral and ethical dilemmas surrounding the issue.
  • Analyze the ethical issue and compare them to the state health laws and regulations in your state.
  • Outline the process of ethical decision making you would use to address this ethical dilemma.

Note: Be sure to use the Practicum Journal Template, located in this week’s Learning Resources.

Conduct your community observation and take notes if necessary.

NUR206: Week 9 Assignment Page 1 `

© 2018

Assignment: Community Resource

Assignment Overview In this assignment, you will conduct a discrete and respectful community observation of an Alcoholics Anonymous meeting. Locate and attend a local Alcoholics Anonymous meeting in your area. Explain to the meeting leader that you are a nursing student studying community resources. Write a two page summary of your experience describing the clientele (be sure to respect members’ anonymity), meeting, and topics. Apply your nursing knowledge to evaluate the quality and effectiveness of this particular resource.

Assignment Details:

Perform the following tasks: • Complete the reading assignment and the interactive lesson before attempting this

assignment. • Attend a local Alcoholics Anonymous meeting in your area

o Introduce yourself to the meeting leader and explain to him/her that you are a nursing student who is studying community resources.

o Conduct your community observation and take notes if necessary. • Write a two-page summary of your experience

o Describe the clientele (respecting members’ anonymity), meeting, and topics. o Apply your nursing knowledge to evaluate the quality and effectiveness of this particular

resource. o Use APA format.

• Include the proper file naming convention: NUR206_wk9_assn_jsmith_mmddyyy.

Grading:

Criteria Excellent (3pts) Good (2pts) Needs Improvement (1pt)

Pts.

Content -Includes description of clientele. -Describes meeting. -Evaluates quality and effectiveness using nursing knowledge.

-Includes description of clientele. -Vaguely describes meeting. -Non-specific evaluation.

-Vague description of clientele. -Missing description of meeting. -Does not apply nursing knowledge in evaluation.

Organization -Well organized into separate headings. -Main points are clearly identified. -Paragraphs transition well.

-Somewhat organized. -Main points are clear. -Occasional issues with flow and transitions.

-Disorganized. -Main points are unclear -Difficult to follow or illogical.

Spelling/grammar/ APA

-No spelling or grammar errors. -APA format is correct. -Appropriate length

-Some spelling or grammar errors. -APA format is correct. -Appropriate length.

-Many spelling or grammar errors. -Incorrect APA format. -Inappropriate length.

Instructor:

Total score

  • Assignment Overview
  • Assignment Details:

What are some of the nutritional challenges for emerging populations? What roles do nutritional deficiency and nutritional excess play in disease? 

 Discuss why nutrition is a central component in health promotion. What are some of the nutritional challenges for emerging populations? What roles do nutritional deficiency and nutritional excess play in disease?

DISCUSSION!!

 Discussion: Diversity and Health Assessments

Discussion: Diversity and Health Assessments
May 2012, Alice Randall wrote an article for The New York Times on  the cultural factors that encouraged black women to maintain a weight  above what is considered healthy. Randall explained�from her  observations and her personal experience as a black woman�that many  African-American communities and cultures consider women who are  overweight to be more beautiful and desirable than women at a healthier  weight. As she put it, �Many black women are fat because we want to be�  (Randall, 2012).
Randall�s statements sparked a great deal of controversy and debate;  however, they emphasize an underlying reality in the healthcare field:  different populations, cultures, and groups have diverse beliefs and  practices that impact their health. Nurses and healthcare professionals  should be aware of this reality and adapt their health assessment  techniques and recommendations to accommodate diversity.
In this Discussion, you will consider different socioeconomic,  spiritual, lifestyle, and other cultural factors that should be taken  into considerations when building a health history for patients with  diverse backgrounds. Your Instructor will assign a case study to you for  this Discussion.

CASE STUDY:

EB is a 68-year-old black female who comes in for  follow-up of hypertension. She has glaucoma and her vision has been  worsening during the past few years. She lives alone and is prescribed  four hypertension medications (Hydralazine 50 mg PO Q8H, Metoprolol XL  200 mg PO Q12H, Lisinopril 40 mg PO daily, and HCTZ 25mg PO daily ). She  brings in her medication bottles and she has some medication bottles  from the previous year full of medications. She is missing one  medication she had been prescribed and says she may have forgotten it at  home. Her BP in clinic today is 182/99 with HR of 84.

To prepare:
� Reflect on your experiences as a nurse and on the information  provided in this week�s Learning Resources on diversity issues in health  assessments.
� By Day 1 of this week, you will be assigned a case study by your  Instructor. Note: Please see the �Course Announcements� section of the  classroom for your case study assignment.
� Reflect on the specific socioeconomic, spiritual, lifestyle, and  other cultural factors related to the health of the patient assigned to  you.
� Consider how you would build a health history for the patient.  What questions would you ask, and how would you frame them to be  sensitive to the patient�s background, lifestyle, and culture? Develop  five targeted questions you would ask the patient to build his or her  health history and to assess his or her health risks.
� Think about the challenges associated with communicating with  patients from a variety of specific populations. What strategies can you  as a nurse employ to be sensitive to different cultural factors while  gathering the pertinent information?
Post an explanation of the specific socioeconomic, spiritual,  lifestyle, and other cultural factors associated with the patient you  were assigned.

Explain the issues that you would need to be sensitive to  when interacting with the patient, and why.

Provide at least five  targeted questions you would ask the patient to build his or her health  history and to assess his or her health risks.

NOTE: THIS IS THE LINK TO DOWNLOAD THE BOOK

https://www.sendspace.com/file/wd7quh

Describe the current health status of this group. How do race and ethnicity influence health for this group?

Select an ethnic minority group that is represented in the United States (American Indian/Alaskan Native, Asian American, Black/African American, Hispanic/Latino, Native Hawaiian, or Pacific Islander). Using health information available from Healthy People, the CDC, and other relevant government websites, analyze the health status for this group.

In a paper of 1,000-1,250 words, compare and contrast the health status of your selected minority group to the national average. Include the following:

  1. Describe the ethnic minority group selected. Describe the current health status of this group. How do race and ethnicity influence health for this group?
  2. What are the health disparities that exist for this group? What are the nutritional challenges for this group?
  3. Discuss the barriers to health for this group resulting from culture, socioeconomics, education, and sociopolitical factors.
  4. What health promotion activities are often practiced by this group?
  5. Describe at least one approach using the three levels of health promotion prevention (primary, secondary, and tertiary) that is likely to be the most effective in a care plan given the unique needs of the minority group you have selected. Provide an explanation of why it might be the most effective choice.
  6. What cultural beliefs or practices must be considered when creating a care plan? What cultural theory or model would be best to support culturally competent health promotion for this population? Why?

Cite at least three peer-reviewed or scholarly sources to complete this assignment. Sources should be published within the last 5 years and appropriate for the assignment criteria and public health content.

Prepare this assignment according to the guidelines found in the APA Style Guide, located in the Student Success Center. An abstract is not required.

This assignment uses a rubric. Please review the rubric prior to beginning the assignment to become familiar with the expectations for successful completion.

Is there a difference between “common practice” and “best practice”?

Is there a difference between “common practice” and “best practice”?

When you first went to work for your current organization, experienced colleagues may have shared with you details about processes and procedures. Perhaps you even attended an orientation session to brief you on these matters. As a “rookie,” you likely kept the nature of your questions to those with answers that would best help you perform your new role.

Over time and with experience, perhaps you recognized aspects of these processes and procedures that you wanted to question further. This is the realm of clinical inquiry.

Clinical inquiry is the practice of asking questions about clinical practice. To continuously improve patient care, all nurses should consistently use clinical inquiry to question why they are doing something the way they are doing it. Do they know why it is done this way, or is it just because we have always done it this way? Is it a common practice or a best practice?

In this Assignment, you will identify clinical areas of interest and inquiry and practice searching for research in support of maintaining or changing these practices. You will also analyze this research to compare research methodologies employed.

To Prepare:

  • Review the Resources and identify a clinical issue of interest that can form the basis of a clinical inquiry.
  • Based on the clinical issue of interest and using keywords related to the clinical issue of interest, search at least four different databases in the Walden Library to identify at least four relevant peer-reviewed articles related to your clinical issue of interest.
  • Review the results of your peer-reviewed research and reflect on the process of using an unfiltered database to search for peer-reviewed research.
  • Reflect on the types of research methodologies contained in the four relevant peer-reviewed articles you selected.

Part 1: An Introduction to Clinical Inquiry

Create a 4- to 5-slide PowerPoint presentation in which you do the following:

  • Identify and briefly describe your chosen clinical issue of interest.
  • Describe how you used keywords to search on your chosen clinical issue of interest.
  • Identify the four research databases that you used to conduct your search for the peer-reviewed articles you selected.
  • Provide APA citations of the four peer-reviewed articles you selected.

Part 2: Identifying Research Methodologies

After reading each of the four peer-reviewed articles you selected, use the Matrix Worksheet template to analyze the methodologies applied in each of the four peer-reviewed articles. Your analysis should include the following:

  • The full citation of each peer-reviewed article in APA format.
  • A brief (1-paragraph) statement explaining why you chose this peer-reviewed article and/or how it relates to your clinical issue of interest, including a brief explanation of the ethics of research related to your clinical issue of interest.
  • A brief (1-2 paragraph) description of the aims of the research of each peer-reviewed article.
  • A brief (1-2 paragraph) description of the research methodology used. Be sure to identify if the methodology used was qualitative, quantitative, or a mixed-methods approach. Be specific.
  • A brief (1- to 2-paragraph) description of the strengths of each of the research methodologies used, including reliability and validity of how the methodology was applied in each of the peer-reviewed articles you selected.

Post a description of experiences or observations about how nurse informaticists and/or data or technology specialists interact with other professionals within your healthcare organization.

Nature offers many examples of specialization and collaboration. Ant colonies and bee hives are but two examples of nature’s sophisticated organizations. Each thrives because their members specialize by tasks, divide labor, and collaborate to ensure food, safety, and general well-being of the colony or hive.

Of course, humans don’t fare too badly in this regard either. And healthcare is a great example. As specialists in the collection, access, and application of data, nurse informaticists collaborate with specialists on a regular basis to ensure that appropriate data is available to make decisions and take actions to ensure the general well-being of patients.

In this Discussion, you will reflect on your own observations of and/or experiences with informaticist collaboration. You will also propose strategies for how these collaborative experiences might be improved.

To Prepare:

  • Review the Resources and reflect on the evolution of nursing informatics from a science to a nursing specialty.
  • Consider your experiences with nurse Informaticists or technology specialists within your healthcare organization.

By Day 3 of Week 3

Post a description of experiences or observations about how nurse informaticists and/or data or technology specialists interact with other professionals within your healthcare organization. Suggest at least one strategy on how these interactions might be improved. Be specific and provide examples. Then, explain the impact you believe the continued evolution of nursing informatics as a specialty and/or the continued emergence of new technologies might have on professional interactions.